RegistrierenRegistrieren   LoginLogin   FAQFAQ    SuchenSuchen   
Perle auf geradem rotierenden Draht
 
Neue Frage »
Antworten »
    Foren-Übersicht -> Mechanik
Autor Nachricht
doeka



Anmeldungsdatum: 10.11.2016
Beiträge: 58

Beitrag doeka Verfasst am: 04. Jan 2017 22:57    Titel: Perle auf geradem rotierenden Draht Antworten mit Zitat

Meine Frage:
Huhu, ich denke, ich stehe mal wieder komplett auf dem Schlauch. Ich muss auch zugeben, dass ich das Vorgehen bei dieser Problematik nicht ganz verstanden habe. Unser Prof ist so bezeichnungswütig, dass ich große Mühe habe die Formeln nachzuvollziehen, die er so ankritzelt. Dadurch stockt das Lernen bei mir gewaltig.

Jedenfalls soll ich diese schicke Aufgabe lösen, aber ich weiß einfach nicht wie (ich scheitere schon an der ersten Teilaufgabe)

Der Stab soll nach meinem Verständnis mit einem Ende im Ursprung verankert sein und um diesen in der x,y-Ebene rotieren.
Der Betrag des Radialvektors muss zeitabhängig sein, ebenso wie die Winkelgeschwindigkeit

Wir haben die Lagrange-Formeln noch nicht so wirklich behandelt, wir sollen für die Bewegungsgleichung vom D'Alembert-Prinzip ausgehen.

Meine Ideen:
Da ich zuerst nicht auf die Zwangsbedingungen gekommen bin, habe ich nachgeschaut.




Weiter bin ich noch nicht, besser gesagt nicht mehr. Ich habe dann alles verworfen, weil ich mich total verwirrt habe.

Ich weiß, dass es sinnvoll ist in Polarkoordinaten zu arbeiten. Ich kam jedoch immer darauf, dass die Bahngeschwindigkeit Null wird. unglücklich



TM1617_09.pdf
 Beschreibung:

Download
 Dateiname:  TM1617_09.pdf
 Dateigröße:  75.45 KB
 Heruntergeladen:  445 mal

doeka



Anmeldungsdatum: 10.11.2016
Beiträge: 58

Beitrag doeka Verfasst am: 04. Jan 2017 23:07    Titel: Antworten mit Zitat

Was trotzdem nochmal rausgearbeitet habe:



Z ist die Zwangskraft. Dann habe ich mit dem D'Alembert-Prinzip diese Formel bekommen:



Ist das soweit in Ordnung? Ich denke mir, dass F die Zentrifugalkraft sein müsste, also jene Kraft, die dafür sorgt, dass die Kugel sich "nach außen" bewegt.

Ich habe dann versucht die Zentrifugalkraft einzusetzen, aber es kam nix bei rum unglücklich
franz



Anmeldungsdatum: 04.04.2009
Beiträge: 11583

Beitrag franz Verfasst am: 05. Jan 2017 02:01    Titel: Antworten mit Zitat

Die rotierende Gerade in Zylinderkoordinaten
Lagrange I



Wegen der NB
wo ein exponentieller Ansatz zu führt.
Ab hier wird es interessant, zum Beispiel bei einem Start zum Zentrum hin ...
Die Zwangskraft dient übrigens (aus Sicht mitrotierender Beobachter)
dazu, die Corioliskraft zu kompensieren.
doeka



Anmeldungsdatum: 10.11.2016
Beiträge: 58

Beitrag doeka Verfasst am: 05. Jan 2017 10:02    Titel: Antworten mit Zitat

Huhu danke dir, ich war wohl wirklich wieder blind.

Aber nur nochmal zum Verständnis:

Du hast aus den Komponenten der Vektoren ein Gleichungssystem erhalten, oder?
Und für F wurde hier tatsächlich die Zentrifugalkraft eingesetzt.

Wie kommt man aber auf die Komponenten für Z?

Was ich auch nicht verstehe, ist das mit der Corioliskraft. Die Perle rotiert doch ebenfalls mit dem System (Massenpunkt gleitet in hohlem Stab). Wenn ich als Beobachte auch mitrotiere, dann müsste ich doch - ich stell mir das jetzt so vor, dass ich mich quasi mitdrehe und von oben draufschaue - eine geradlinige Bewegung der Perle beobachten. Da ich mich selbst mit gleicher Winkelgeschwindigkati ja mit der Perle auf gleiche Weise drehe, könnte man auch annehmen, unser Bezugssystem ruhe. Oder irre ich mich da?

Würde die Corioliskraft nicht dann auftreten, wenn sich die Perle vom Draht losgelöst geradlinig bewegen würde?
TomS
Moderator


Anmeldungsdatum: 20.03.2009
Beiträge: 18200

Beitrag TomS Verfasst am: 05. Jan 2017 10:37    Titel: Antworten mit Zitat

Ich würde eine andere Formulierung mittels Lagrangefunktion wählen.

Letztere ist für das freie Problem gegeben durch



Die Bedingung, dass sich der Massenpunkt in der Röhre bewegt, kann mittels



implementiert werden; das letzte Gleichheitszeichen gilt speziell für konstante Winkelgeschwindigkeit.

Die um die Zwangsbedingung erweiterte Lagrangefunktion mit Lagrangemultiplikator lambda lautet nach Einsetzen



Die Bewegungsgleichungen einschließlich der für lambda folgen wie üblich als Euler-Lagrange-Gleichungen für eine explizit zeitabhängige Lagrangefunktion. Die Gleichung für den Lagrangemultiplikator lambda reproduziert dabei die Zwangsbedingung C.

Der Vorteil ist zum Einen, dass du die Zwangskraft nun einfach berechnen kannst. Die Komponenten ergeben sich automatisch aus dem Formalismus. Zum Anderen kannst du Koordinatentransformationen ins mitrotierende System für die Lagrangefunktion einfacher durchführen.

_________________
Niels Bohr brainwashed a whole generation of theorists into thinking that the job (interpreting quantum theory) was done 50 years ago.
doeka



Anmeldungsdatum: 10.11.2016
Beiträge: 58

Beitrag doeka Verfasst am: 05. Jan 2017 15:19    Titel: Antworten mit Zitat

Danke TomS, das ist auch sehr sinnvoll. Das Ding ist eben nur, dass wir das laut Aufgabenstellung mit dem D'Alembertschen Prinzip so machen sollen.

Die Lagrange-Funktion haben wir heute erst gehabt, die Euler-Lagrange- Gleichung auch.
Ich kann mir schon denken, dass sicher irgendeine Bemerkung kommt, von wegen "So sollte das aber nicht gemacht werden" - sowas mag ich ja eigentlich gar nicht, wenn es auch auf andere Weise möglich ist. Aber so sind wohl die Spielregeln während des Studiums.

Die Winkelgeschwindigkeit soll in der ersten Teilaufgabe nicht konstant sein, aber müsste die Zwangsbedingung dann nicht trotzdem so lauten? Nur eben, dass von t abhängt?
franz



Anmeldungsdatum: 04.04.2009
Beiträge: 11583

Beitrag franz Verfasst am: 05. Jan 2017 19:12    Titel: Antworten mit Zitat

Hallo doeka,

die Zwangskräfte realisieren quasi die Zwangs- oder Nebenbedingungen:
a) Keine Einschränkung für r, keine von außen eingeprägte Radialkraft
b) Eine Zwangskraft zur Realisierung von . Die oben notierte Corioliskraft muß noch um einen Term (Bezeicnung?) ergänzt werden wg der beschleunigten Rotation.
c) ist klar.

i) Wg der Bewegung auf der Stange bleibt von den ganzen Zirkus nur übrig.

ii) Bei greift der schon erwähnte exponentielle Ansatz (formal ähnlich dem Oszillator, aber wegen der reellen Lösungen keine Schwingung). Die Lösung ist noch nicht komplett.

iii) Fehlt noch.

Zum mitrotierenden Beobachter: Der stellt a) die Zentrifugalkraft fest (die Perle wird nach außen geschossen) und b) bei Bewegung der Perle eine seitliche Kraft (Corioliskraft + Zusatzterm wegen beschleunigter Rotation), die jedoch in unserem Fall vom Rohr kompensiert wird. Natürlich ruht dieser Beobachter in seinem System, es passiert bloß mehr als sonst / in Inertialsystemen.

Sollte die Perle sich frei bewegen, wird der rotierende Beobachter weiterhin dies seitlichen Kräfte registrieren und eine entsprechende Bewegung.
doeka



Anmeldungsdatum: 10.11.2016
Beiträge: 58

Beitrag doeka Verfasst am: 09. Jan 2017 19:44    Titel: Antworten mit Zitat

OK, ich denke ich hab es nun verstanden. Dankeschön smile
Neue Frage »
Antworten »
    Foren-Übersicht -> Mechanik